Understanding Complex Variable Power Series

Click For Summary
The discussion revolves around two complex variable power series problems related to convergence and coefficients. For part (a), the user struggles to understand why all coefficients must be zero if the function f(z) equals zero within a certain radius, prompting a suggestion to apply Taylor's Theorem. In part (b), the user questions the necessity of proving that coefficients of two equal series must match, suggesting a straightforward comparison seems sufficient. However, hints indicate that a formal proof is required by combining the series and applying the result from part (a). Overall, the thread emphasizes the need for deeper mathematical reasoning in both problems.
student85
Messages
138
Reaction score
0
Hi all, I'm having trouble understading this problem I got for homework. They're actually two problems in one, (a) and (b)... Any comment you can give me for understaing this will REALLY help. This is due Thursday, so please if anyone knows anything about this, can you share it with me? Thank you.

Homework Statement


a) If f(z) = \sum an(z-z0)n has a radius of convergence R>0 and if f(z)=0 for all z,|z-z0| < r \leq R, show that a0=a1=...=0.

b) If F(z) = \sum an(z-z0)n and G(z) = \sum bn(z-z0)n are equal on some disc |z-z0|< r, show that an = bn for all n.

NOTE: All sums go from n=0 to infinity.

Homework Equations




The Attempt at a Solution



Ok first of all I don't really get (a). It says f(z) is equal to 0 for all z, so obviously all a's must be equal to 0, no? Can someone please tell me what I am missing here? I typed the wording for the problem exactly as it is in the textbook.
Then for (b), again I don't see the point in the problem I mean if F and G are equal in the disc, then obviously all a's and b's are equal because each is the coefficient to a specific power of z, so a0 must be equal to b0, a1 = b1, and so on. But there must be more to these problems, there must be some mathematical proof or something I must do. Can someone shed some light on me? Thanks again.
 
Physics news on Phys.org
Hints
(a) Use the same Taylor's Theorem from real analysis (works the same way for analytic functions)

(b) Subtract F and G from each other and compose the series into one series. Use the result from (a) so that a-b=0 for all n.
 
Question: A clock's minute hand has length 4 and its hour hand has length 3. What is the distance between the tips at the moment when it is increasing most rapidly?(Putnam Exam Question) Answer: Making assumption that both the hands moves at constant angular velocities, the answer is ## \sqrt{7} .## But don't you think this assumption is somewhat doubtful and wrong?

Similar threads

  • · Replies 3 ·
Replies
3
Views
2K
  • · Replies 2 ·
Replies
2
Views
2K
  • · Replies 2 ·
Replies
2
Views
2K
Replies
3
Views
2K
Replies
6
Views
2K
  • · Replies 1 ·
Replies
1
Views
2K
  • · Replies 6 ·
Replies
6
Views
2K
  • · Replies 22 ·
Replies
22
Views
3K
  • · Replies 2 ·
Replies
2
Views
3K
Replies
7
Views
1K